The Student Room Group

Matrix question

Screenshot_20160513-234201.jpg

Part a I found fine thanks to earlier help but part b is confusing.

I tried to use determinant to guide me but to no avail.

How do I do this? It's only worth one mark so I feel like I'm missing something easy.
Reply 1
Original post by Ravster
Screenshot_20160513-234201.jpg

Part a I found fine thanks to earlier help but part b is confusing.

I tried to use determinant to guide me but to no avail.

How do I do this? It's only worth one mark so I feel like I'm missing something easy.


If you multiply a 2x2 matrix by the 2x2 identity matrix you end up with the same matrix.

As A^n is just doing the transformation n times, what it's really asking you is how many times do you need to carry out the transformation to get back to where you started.
Original post by Ravster
Screenshot_20160513-234201.jpg

Part a I found fine thanks to earlier help but part b is confusing.

I tried to use determinant to guide me but to no avail.

How do I do this? It's only worth one mark so I feel like I'm missing something easy.

If you have some coordinates (xy)\begin{pmatrix}x \\ y \end{pmatrix} and you multiply it by the identity matrix I, you will get the same thing: (xy)\begin{pmatrix}x \\ y \end{pmatrix}.

As A is the rotation about the origin by certain angle, how many times will it have rotate by that angle to get to its original position k360k360^{\circ}

Edit: Zacken has pointed out that you would have to get to multiple of 360360^{\circ} which would be the LCM of the angle you found in part(a) and 360.
(edited 7 years ago)
Reply 3
Original post by Ravster
Screenshot_20160513-234201.jpg

Part a I found fine thanks to earlier help but part b is confusing.

I tried to use determinant to guide me but to no avail.

How do I do this? It's only worth one mark so I feel like I'm missing something easy.

Think about what the angle of rotation A represents and think about how many multiples of that rotation you need to get to a multiple of 360 degrees, which will be the identity matrix.

Remember that if A represents a rotation of α\alpha then A2A^2 is a rotation of α\alpha followed by a rotation of α\alpha for a total rotation of 2α2\alpha. In general here AnA^n is a rotation of nαn\alpha degrees. So you need to find a nn s,t nαn\alpha is a multiple of 360.
Reply 4
Original post by shivtek
Part was 135 I think. Or 45. I can't remember. I is the identity as you know so what it's asking is how many times does one apply A to get back to the start. So you do 360/(135 or 45). I got 8/3 I think


Original post by Kvothe the arcane
If you have some coordinates (xy)\begin{pmatrix}x \\ y \end{pmatrix} and you multiply it by the identity matrix I, you will get the same thing: (xy)\begin{pmatrix}x \\ y \end{pmatrix}.

AA represents a transformation and it's saying that A×A××A×....×An times=X\mathrm{\underbrace{A \times A \times \times A \times .... \times A}_{n \ times}=X}

As A is the rotation about the origin by certain angle, how many times will it have rotate by that angle to get to its original position 360360^{\circ}


I'm going to have to butt in here and say that you want a natural nn; so n=83n=\frac{8}{3} is incorrect. You don't want to rotate to get to 360360^{\circ} but rather, you want to rotate to get to a natural multiple of 360360^{\circ}. Which is an important point here.
Original post by Zacken
...


That makes sense. You can't have a fraction of a multiplication. It would have made sense to evaluate a.
Thanks for pointing it out.
(edited 7 years ago)
Reply 6
Original post by Zacken
I'm going to have to butt in here and say that you want a natural nn; so n=83n=\frac{8}{3} is incorrect. You don't want to rotate to get to 360360^{\circ} but rather, you want to rotate to get to a natural multiple of 360360^{\circ}. Which is an important point here.


Away good point I didn't read "positive integer"
So really, raising it to the power of 8 would work, as it would be 0 rotation modulo 360. Thanks for pointing that out!
Reply 7
Original post by shivtek
Away good point I didn't read "positive integer"
So really, raising it to the power of 8 would work, as it would be 0 rotation modulo 360. Thanks for pointing that out!


Yep, 8 is the only correct answer here since it specifies nn being the smallest positive integer satisfying the condition. Right on with the rotation 0 modulo 360!

Quick Reply

Latest